Đến nội dung

Kamii0909 nội dung

Có 155 mục bởi Kamii0909 (Tìm giới hạn từ 28-04-2020)



Sắp theo                Sắp xếp  

#660309 $2^{n}a+b$

Đã gửi bởi Kamii0909 on 02-11-2016 - 14:33 trong Số học

Từ đề bài ta có tồn tại dãy số nguyên không âm $(x_n)_{n\ge 1}$ sao cho $a.2^n+b=x_n^2 \Rightarrow x_n=\sqrt{a.2^n+b}$ 
Khi đó ta có $2x_n-x_{n+2}=\frac{3b}{\sqrt{a.2^{n+2}+b}+\sqrt{a.2^{n+2}+b}}$ 
Suy ra $lim_{n \rightarrow +\infty}(2x_n-x_{n+2})=0$ mà dãy $\{2x_n-x_{n+2}\}$ nguyên nên tồn tại $k_0 \in \mathbb{N^*}$ để mà 
$2x_n-x_{n+2}=0,\forall n \ge k_0$ hay $2x_n=x_{n+2},\forall n \ge k_0$ 
$\Leftrightarrow 2\sqrt{a.2^n+b}=\sqrt{a.2^{n+2}+b},\forall n \ge k_0 \Leftrightarrow b=0
Do đó $a.2^n$ là số chính phương với mọi số nguyên không âm $n$. Hiển nhiên ta phải có $a=0$ (đpcm)

Đề ra đâu có $b=0$ đâu,chỉ $a=0$ mà




#673878 Chứng minh $3(a+b+c)\geq \sqrt{8a^2+1}+\sqrt...

Đã gửi bởi Kamii0909 on 10-03-2017 - 17:40 trong Bất đẳng thức và cực trị

KMTTQ, $a \geq b \geq c$
Khi đó dễ cmr $a - \frac{1}{a} \geq b-\frac{1}{b} \geq c - \frac{1}{c}$
Và $3+ \sqrt{8+ \frac{1}{a^2}} \leq 3+ \sqrt{8+ \frac{1}{b^2}} \leq 3+\sqrt{8+\frac{1}{c^2}}$
Bđt cần cm tương đương với
$\sum \frac{a-\frac{1}{a}}{3+ \sqrt{8+\frac{1}{a^2}}} \geq 0$
Áp dụng bđt Cheybershev kết hợp điều kiện ta có đpcm.



#712215 $f(x^2+yf(x))=xf(f(x))+f(x)f(y)$

Đã gửi bởi Kamii0909 on 09-07-2018 - 08:21 trong Phương trình hàm

Tìm tất cả $f: \mathbb{R} \rightarrow \mathbb{R}$ thỏa mãn

$f(x^2+yf(x))=xf(f(x))+f(x)f(y)$ với mọi $x,y \in \mathbb{R}$




#667457 $F,M,Y,Z$ đồng viên

Đã gửi bởi Kamii0909 on 07-01-2017 - 14:37 trong Hình học

Bài này không khó, chắc lấy ý tưởng từ bài Trường Đông năm nay.
$\textbf{Lời giải}$
Kẻ $CH$ cắt $(O)$ tại $K$. $FX$ cắt $AK$ tại $N$.
Dễ thấy $FX=FN$ nên áp dụng định lý con bướm đảo vào tứ giác nội tiếp $AKBC$ tâm $O$ có $OF \perp FX$.
Phần sau dễ rồi.



#689907 $f(xy)=max\left \{ f(x+y), f(x).f(y) \right \}$

Đã gửi bởi Kamii0909 on 08-08-2017 - 15:25 trong Phương trình hàm

Bổ đề: Xét $a \geq 0$ và $g: \mathbb{R} \rightarrow \mathbb{R}$ thoả mãn $g(x)=x - \dfrac{a}{x}$ thì $g$ toàn ánh.(Không chứng minh) 

 

$P(x,y) : f(xy) = max \left \{ f(x+y),f(x)f(y) \right \}$

$P \left(x, \dfrac{-a}{x} \right) : f(-a)= max \left \{ f \left( x- \dfrac{a}{x} \right), f(x)f \left( \dfrac{-a}{x} \right) \right \}$

Tức là $f(-a) \geq f \left( x- \dfrac{a}{x} \right) (1)$
Từ đây do tính toàn ánh của $ x- \dfrac{a}{x}$ nên $f(-a) \geq f(x), \forall a \geq 0, x \in \mathbb{R}$

Hay $f(-y) \geq f(x), \forall y \geq 0, \forall x \in \mathbb{R} $

Từ đây $f(-y) \geq f(-x), \forall x,y \geq 0$ suy ra $f(x)=f(0)=C, \forall x \leq 0$ 

Ngoài ra, $f(x) \leq C, \forall x \in \mathbb{R}$

Xét $x \geq 0$

$P(-x,-1) : f(x)= max  \left \{ f(-x-1), f(-x)f(-1) \right \} = max \left \{ C, C^2  \right \}$

$\Rightarrow C \geq C^2 \Leftrightarrow 0 \ leq C \leq 1$

Mặt khác, với mọi $C \in [0,1]$ thì $C \geq C^2$ nên $f(x)=C, \forall x$




#659194 cho đa thức f(x) tìm dư của phép chia f(x) cho (x-1)(x-2)

Đã gửi bởi Kamii0909 on 24-10-2016 - 18:21 trong Bất đẳng thức và cực trị

Đặt $x+y=a$ (cho đẹp) và $xy=b$
P=$(x+y)^3-3xy(x+y)+2xy$
$P=a^3-3ab+2b=a^3+b(2-3a)$
Ta có $4xy=4b=(x+y)^2+(x-y)^2=a^2+(x-y)^2$
Đến đây chắc em làm được. Chỉ việc xét min max của $(x-y)^2$ mà x,y nguyên nên công việc này rất dễ.



#691650 $(x^{2}+y+f(y))=f(x)^{2}+a, \forall x,y \i...

Đã gửi bởi Kamii0909 on 27-08-2017 - 02:22 trong Phương trình hàm

Chuẩn phải là $ P(x,y): f(x^2+y+f(y))=f^2(x)+ay, \forall x,y \in \mathBB{R}$
Nếu $a=0$ dễ chỉ ra 2 hàm $f(x) =0,f(x)=1$ thoả mãn. 
Xét $a \neq 0$ thì ta có $f$ toàn ánh. 

Nếu $f(a)=0$ ta chỉ ra $a=0$
Thật vậy, ta có
$P(b,y)-P(-b,y): f(b)=f(-b)=0$
$P(0,b)+P(0,-b): f(b)+f(-b)=2f^2(0)=0$
Tức là $f(0)=0$, từ đó $P(0,b): ab=0 \Leftrightarrow b=0$

 

Chứng minh $f(x^2)=bx^2, b \geq 0, b^2+b=a$
$P(x,0):f(x^2)=f^2(x)$
Từ đây $f$ đơn ánh trên từng khoảng $(0,+ \infty)$ và $(- \infty,0)$ và $f(x) \geq 0, \forall x \geq 0$
Từ đó $f(x^2)+x^2 \geq 0,\forall x$
$P(0,x):f(x+f(x))=ax$
$P( \sqrt{x^2+f(x^2)},y^2): f(x^2+y^2+f(x^2)+f(y^2))=a(x^2+y^2)$
Và $P(0,x^2+y^2): f(x^2+y^2+f(x^2+y^2))=a(x^2+y^2)$
Do $f$ đơn ánh trên $(0, +\infty)$ nên $f(x+y)=f(x)+f(y),\forall x,y \geq 0$
Kết hợp với $f(x) \geq 0,\forall x \geq 0$ 
Và xét $f$ trên $(0, +\infty)$ dễ thu được $f(x)=bx,\forall x \geq 0(b > 0)$
Trong đó $b^2+b=a$

Bây giờ ta chứng minh $f(-x^2) \leq 0, \forall x$ 
Giả sử $\exists x,f(-x^2) \geq 0$ 
$P(x,-x^2):f(f(-x^2))=f(x^2)-ax^2=-b^2x^2 <0$
Mà $f(-x^2)>0$ nên $f(f(-x^2)) >0$
Mâu thuẫn, tức là $f(-x^2) \leq 0$

 

$f(x)=-f(-x)$

Ta có $f(x^2)=f(-x)^2=f(x)^2$

Dễ thấy 2 số $x$ và $-x$ khác dấu $ \forall x \neq 0$ nên $f(x)$ và $f(-x)$ cũng khác dấu.

Nói cách khác $f(x) \neq f(-x), \forall x \neq 0$ dẫn tới $f(x)=-f(-x), \forall x$

$f(x^2)=bx^2$ nên $f(x)=bx,\forall x$

Trong đó $b^2+b=a \geq 0$

Với $a>0$ ta tìm được nghiệm dương duy nhất của $b$ là $\dfrac{2a}{1+ \sqrt{1+4a}}$

Kết luận $a>0$




#691649 $(x_n):\left\{\begin{matrix} ...\...

Đã gửi bởi Kamii0909 on 27-08-2017 - 00:36 trong Dãy số - Giới hạn

Chứng minh theo quy nạp rằng $x_n \leq \dfrac{25}{4}- \dfrac{5}{n}, n \geq 1$
Kiểm tra với $n=1,2$ đúng. 
Sử dụng công thức truy hồi, ta cần chỉ ra 
$$x_{n+1}<x_{n}+\dfrac{5}{n^3} \leq \dfrac{25}{4} - \dfrac{5}{n} + \dfrac{5}{n^3} \leq \dfrac{25}{4}- \dfrac{5}{n+1}, \forall n \geq 2$$

$$\Leftrightarrow n^2 \geq n+1,\forall n \geq 2$$

Từ đó ta có đpcm.




#662053 cho a,b,c>0.CMR: $\sum \sqrt{\frac{a}...

Đã gửi bởi Kamii0909 on 15-11-2016 - 19:26 trong Bất đẳng thức và cực trị

1 cách nữa cho bài 2.
Không mất tính tổng quát giả sử $a \geq b \geq c$.
Khi đó $VP \geq \sum \frac{b^2}{b+c}$ và$ VP \geq \sum \frac{c^2}{c+a}$
Từ đó ta đi chứng minh
$\sum \frac{b^2+c^2}{b+c} \geq \sqrt \frac{2011}{2}$
Áp dụng bất đẳng thức Cauchy-Schwarz ta có $2VP \geq \frac{(\sum \sqrt {a^2+b^2})^2 }{2\sum a}$
Dễ dàng suy ra đpcm từ đây



#663371 CHỨNG MINH THẲNG HÀNG

Đã gửi bởi Kamii0909 on 29-11-2016 - 18:04 trong Hình học

Gọi $AI,BI,CI$ cắt $(ABC)$ tại $M,N,P$.
Dễ dàng chứng minh $I$ là trực tâm tam giác $MNP$.
Bổ đề:Cho tam giác $ABC$ trực tâm $H$ nội tiếp $(O)$. Khi đó $\overrightarrow{OH}=\overrightarrow{OA} +\overrightarrow{OB} + \overrightarrow{OC}$.
Áp dụng bổ đề trên với chú ý $I,O$ là tâm ngoại của $MNP$ và $DEF$ và $\overrightarrow{ID} = \frac{r}{R}.\overrightarrow{OM}$ ta dễ có $\overrightarrow{OI}= \frac{R}{r} \overrightarrow{IH}$
Hệ thức trên chứng tỏ $\overline{O,I,H}$.



#667003 $\frac{1}{a^{2}}+\frac{1...

Đã gửi bởi Kamii0909 on 04-01-2017 - 23:21 trong Bất đẳng thức và cực trị

Đặt $(a,b,c) \rightarrow \left( \dfrac{1}{x},\dfrac{1}{y},\dfrac{1}{z} \right)$
Ta biến đổi bđt cần cm về
$$x^2+y^2+z^2+3 \geq 2(\frac{1}{x}+\frac{1}{y}+\frac{1}{z})\Leftrightarrow x^2+y^2+z^2+2xyz+1 \geq 2(xy+yz+xz)$$
Bất đẳng thức cuối quen thuộc.



#661756 Bài toán về tứ giác toàn phần

Đã gửi bởi Kamii0909 on 13-11-2016 - 12:03 trong Hình học

Mình vẽ hình chả thấy đồng viên ,chỉ thấy thẳng hàng : 

Từ $O$ hạ đường vuông góc đến $PQ$ nghĩa là đường kẻ từ $O$ đến vuông với $PQ$ đi qua tâm $(ABCD)$ ,hay $R$ là điểm Miquel của tứ giác $ABCD.PQ$ từ đó có $(ABQR),(ADPR)$ là các tứ giác nội tiếp nên theo Simson thì các điểm $Z,Y,X,T$ thẳng hàng

ABCD có nội tiếp đâu bạn.




#664444 Tìm các số nguyên tố p thỏa:$p^2+23$ có đúng 6 ước chung.

Đã gửi bởi Kamii0909 on 12-12-2016 - 11:10 trong Số học

Nếu $p=2$ thì $A=p^2+23=27(KTM)$. Nếu $p=3$ thì $A=32(TM)$
Nếu $p>3$ thì $4,3|p^2-1$ nên $4,3|p^2+23$. Dễ thấy $p$ sẽ có ít nhất các ước $1,2,3,4,6,12,p^2+23$(vô lý)
Vậy $p=3$ là giá trị duy nhất.



#673314 $(a+b)^2(b+c)^2(a+c)^2\geq abc(a+2b+c)(a+2c+b)(2a+b+c)$

Đã gửi bởi Kamii0909 on 03-03-2017 - 01:10 trong Bất đẳng thức - Cực trị

Xét $3(a+b)^2(b+c)^2(c+a)^2-3abc(a+b+2c)(b+c+2a)(c+a+2b)=q^2(p^2-3q)+(3q+2p^2)(q^2-3pr) \geq 0$




#668675 Chứng minh di chuyển trên 1 đườnng cố định

Đã gửi bởi Kamii0909 on 17-01-2017 - 17:10 trong Hình học

Cách của mình. Ta sẽ nhắc lại không chứng minh một số kết quả cơ bản sau.
1. Cho tam giác $ABC$ nội tiếp $(O)$. Tiếp tuyến tại $B,C$ của $(O)$ cắt nhau tại $P$. Khi đó $AP$ là đường đối trung đỉnh $A$ của tam giác $ABC$.
2. Trong tam giác $ABC$, đường đối trung đỉnh $A$ cắt $BC$ tại $D$. Khi đó $\frac{DB}{DC}= \frac{AB^2}{AC^2}$
Để xử lí cho gọn, đẹp trước hết ta đi chứng minh bổ đề sau
$\textbf{Bổ đề}$ Cho tam giác $ABC$. Đường đối trung đỉnh $A$ cắt $BC$ tại $Q$. $O,I,J$ lần lượt tâm đường tròn ngoại tiếp tam giác $ABC,AQB,AQC$. Khi đó $OA$ chia đôi $IJ$.
$\textbf{Chứng minh}$
Kẻ $AH,AM$ là đường cao và là trung tuyến của tam giác $ABC$.
Dễ thấy phép vị tự quay tâm $A$ biến $\Delta AIJ \rightarrow \Delta ABC$ biến $H \rightarrow Q$. Mặt khác cũng có $\angle HAM= \angle QAO$ nên biến $ AM \rightarrow AO$. Lại có $AM$ là trung tuyến trong tam giác $ABC$ nên $AO$ là trung tuyến trong tam giác $AIJ$.

$\textbf{Quay lại bài toán}$
Qua $Q$ kẻ đường thẳng $\parallel BC$ cắt $AB,AC$ tại $E,F$. Gọi $I,J$ là tâm đường tròn ngoại tiếp các tam giác $AQE,AQF$.
Theo bổ đề $OA$ chia đôi $IJ$. Ta sẽ chứng minh rằng $O_{1}I=O_{2}J$.
Thật vậy, có $\Delta AIO_{1} \sim \Delta QEB, \Delta AJO_{2} \sim \Delta QFC$ nên ta thu được biến đổi sau.
$$\dfrac{IO_{1}}{JO_{2}}= \frac{IO_{1}}{IA}\cdot \frac{IA}{JA} \cdot \frac{JA}{JO_{2}} = \frac{EB}{QE}\cdot \frac{AB}{AC}\cdot \frac{FQ}{FC}= \frac{AE^2}{AF^2} \cdot \frac{QF}{QE}=1$$
Vậy ta có điều phải chứng minh.



#658042 Tìm giá trị nhỏ nhất $\sum \frac{x}{xy+1}...

Đã gửi bởi Kamii0909 on 16-10-2016 - 11:22 trong Bất đẳng thức và cực trị

Cho x,y,z  dương ;$x+y+z=1$

Tìm Min

$\frac{x}{xy+1}+\frac{y}{yz+1}+\frac{z}{zx+1}$

Giả sử $x\geq y\geq z$

Theo bất đẳng thức Cheybershev ta có 

$\sum \frac{x}{xy+1}\geq \frac{\sum x}{3}\sum \frac{1}{xy+1}\geq \frac{3}{\sum xy+3}\geq \frac{3}{\frac{1}{3}\left ( \sum x \right )^{2}+3}=\frac{9}{10}$




#658826 Tìm giá trị nhỏ nhất $\sum \frac{x}{xy+1}...

Đã gửi bởi Kamii0909 on 22-10-2016 - 21:22 trong Bất đẳng thức và cực trị

Cheybershev có vẻ khó ra hoàn chỉnh
Mình xin đính chính lại cách khác hay hơn 

$\sum \frac{x}{xy+1}=\sum x -\sum \frac{x^{2}y}{xy+1}$

Theo AM-GM 10 số và Holder(đoạn này số siếc loằng ngà loằng ngoằng nên cho phép mình bỏ )
Min=9/10




#658283 Hỏi có thể khẳng định mỗi số trong 2013 số đã cho lớn hơn 3000 hay không?

Đã gửi bởi Kamii0909 on 18-10-2016 - 13:41 trong Số học

Cho 2013 số tự nhiên đôi một khác nhau và khác 0. Biết rằng tổng của 1007 số bất kì luôn lớn hơn tổng của 1006 số còn lại cộng với 2012. Hỏi có thể khẳng định mỗi số trong 2013 số đã cho lớn hơn 3000 hay không?

Bài này nên đưa vào topic Tổ hợp chứ nhỉ :3 
Giải 
Giả sử $a_{1}< a_{2}< ...< a_{2013}$ hay $a_{1}\leq a_{2}-1\leq a_{3}-2\leq ....\leq a_{2013}-2012$

Theo giả thiết ta có 

$\sum_{2}^{1007}a_{i}+a_{1}> \sum_{1008}^{2013}a_{i}+2012\geq \sum_{2}^{1007}a_{i} +1007.1006+2012> \sum_{2}^{1007}+3000$

$\Rightarrow a_{1}> 3000$ từ đó ta có tất cả các số đều lớn hơn 3000




#712214 Tìm hàm f:R R

Đã gửi bởi Kamii0909 on 09-07-2018 - 08:07 trong Phương trình hàm

Tìm hàm $f:\mathbb{R}\rightarrow \mathbb{R}$ thỏa mãn

1.$f(x+f(y+f(x)))=f(x+y)+x$

Kí hiệu $P(x,y)$ thay cho phép thế $x,y$ vào phương trình. 

$P(x,-x):f(x+f(f(x)-x))=x+f(0)$ nên $f$ toàn ánh. 

Do đó tồn tại $a, f(a)=-f(0)$

$P(a,-f(a)):a=0$ nên $f(0)=0$ 

$P(0,x): f(f(x))=f(x)$

$P(x,-x): f(x+f(f(x)-x))=x$

Lấy $f$ 2 vế phương trình này thì $f(x)=x$

 

Tìm hàm $f:\mathbb{R}\rightarrow \mathbb{R}$ thỏa mãn

2.$f(x+f(y+f(x)))=f(x)+x+y$

Kí hiệu $P(x,y)$ thay cho phép thế $x,y$ vào phương trình. 

$P(x,-f(x)):f(x+f(0))=x$ nên $f(x)=x-f(0)$

Tại đây cho $x=0$ thì $f(0)=0$ nên $f(x)=x$




#657053 Chứng minh rằng: $\sum \frac{1}{\sqrt{3+a}}\le...

Đã gửi bởi Kamii0909 on 07-10-2016 - 22:04 trong Bất đẳng thức và cực trị

$\sum \frac{1}{\sqrt{3+a}}\leq \sqrt{3\left ( \sum \frac{1}{a+3 } \right )}$

Ta sẽ chứng minh $\sum \frac{1}{3+a}\leq \frac{3}{4}$

Đổi biến $\left ( a,b,c \right )= \left ( \frac{x}{y},\frac{y}{z},\frac{z}{x} \right )$

Đpcm $\Leftrightarrow \sum \frac{y}{x+3y}\leq \frac{3}{4}\Leftrightarrow \sum \frac{3y}{x+3y}\leq \frac{9}{4}\Leftrightarrow \sum \frac{x}{x+3y}\geq \frac{3}{4}$

Điều này luôn đúng do $\sum \frac{x}{x+3y}= \sum \frac{x^{2}}{x^{2}+3xy}\geq \frac{\left ( \sum x \right )^{2}}{\sum x^{2}+3\sum xy}= \frac{\left ( \sum x \right )^{2}}{\left ( \sum x \right )^{2}+\sum xy}\geq \frac{\left ( \sum x \right )^{2}}{\left ( \sum x \right )^{2}+\frac{1}{3}\left ( \sum x \right )^{2}}=\frac{3}{4}$




#673315 $\sum \frac{a(b+c)}{b^2+bc+c^2}$

Đã gửi bởi Kamii0909 on 03-03-2017 - 01:52 trong Bất đẳng thức - Cực trị

1/CMR:

    a,b,c>0 

          $\sum \frac{a(b+c)}{b^2+bc+c^2}$$\geq 2$

Theo AM-GM $$4(a^2+ab+b^2)(ab+bc+ca) \leq (a+b)^2(a+b+c)^2$$
Bất đẳng thức cần chứng minh đưa về $$\frac{a}{b+c} +\frac{b}{a+c} +\frac{c}{a+b} \geq \frac{(a+b+c)^2}{2(ab+bc+ca)}$$

Hiển nhiên theo C-S. 

Spoiler




#657041 tìm giá trị nhỏ nhất của biểu thức $P=\frac{x^{2}-3x...

Đã gửi bởi Kamii0909 on 07-10-2016 - 21:07 trong Đại số

$P=\frac{\left ( x-1 \right )^{2}-\left ( x-1 \right )+2}{\left ( x-1 \right )^{2}}=\frac{a^{2}-a+2}{a^{2}} \Leftrightarrow a^{2}\left ( 1-P \right )-a+2=0. Ta có \Delta = 1-8\left ( 1-P \right )\geq 0 \Leftrightarrow P\geq \frac{7}{8}$




#693855 $f:\mathbb{Z}_+\rightarrow \mathbb{Z}...

Đã gửi bởi Kamii0909 on 28-09-2017 - 15:38 trong Phương trình hàm

Tìm $f:\mathbb{Z}_+\rightarrow \mathbb{Z}_+$ thỏa mãn:

$f(\frac{f^2(n)}{n})=n$

Ta có $n|f^2(n)$ 

Từ tính chất này cho $n = \dfrac{f^2(n)}{n}$ thì $f^2(n)|n^3$

Đặt $n= p_{1}^{a_1} \cdot p_{2}^{a_2} ....p_{n}^{a_n}$

Trong đó $p_1,p_2,...,p_n$ nguyên tố còn $a_1,a_2,...,a_n \in \mathbb{N*}$

Do $f^2(n)|n^3$ nên $f(n)$ phải có dạng $p_{1}^{b_1} \cdot p_{2}^{b_2}...p_{n}^{b_n}$

Trong đó $b_1,b_2,...b_n \in \mathbb{N}$ và $2b_{i} \leq 3a_{i}$

Do đó $b_{i} \leq a_{i}$ hay $f(n)|n$

Từ đẳng thức này cho $n=\dfrac{f^2(n)}{n}$ thì $n^2|f^2(n)$

Do đó $f(n)=n,\forall n$.

Thử lại TM. 




#672386 Chứng minh $\sum \frac{1}{\sqrt{1+a^2...

Đã gửi bởi Kamii0909 on 22-02-2017 - 14:21 trong Bất đẳng thức - Cực trị

Có thể dùng dồn biến. Chú ý rằng với $ab\leq 1$ ta dễ dàng chỉ ra rằng
$$\frac{1}{\sqrt{1+a^2}}+\frac{1}{\sqrt{1+b^2}} \leq \frac{2}{\sqrt{ab+1}}$$. Thay $ab=\frac{1}{c}$ và xét hàm theo $c$.



#665696 $QF$ tiếp xúc với $(CFM)$

Đã gửi bởi Kamii0909 on 23-12-2016 - 23:54 trong Hình học

Cho hình vuông $ABCD$ nội tiếp $(O)$ và điểm $E$ trên cạnh $CD$. $AE$ cắt $BC$ tại $G$. $BE$ cắt $(O)$ tại $F$. Lấy $M \neq F$ trên $BE$ sao cho $GM=GF$. Gọi $N$ là trung điểm $BC$. $MN$ cắt $CD$ tại $Q$. Chứng minh rằng $QF$ tiếp xúc với $(CFM)$.